LSAT and Law School Admissions Forum

Get expert LSAT preparation and law school admissions advice from PowerScore Test Preparation.

 Administrator
PowerScore Staff
  • PowerScore Staff
  • Posts: 8917
  • Joined: Feb 02, 2011
|
#27403
Complete Question Explanation

Evaluate the Argument-#%. The correct answer choice is (B)

The conclusion of the argument is the first sentence: “George Orwell’s book 1984 has exercised much influence on a great number of this newspaper’s readers.” The basis for this conclusion is that 1984 was the second most named book in a survey about influential books. The argument contains a serious error: just because 1984 came in second in the survey does not mean that “a great number” of readers selected it as influential. To illustrate this proposition, consider the following example:
  • Number of people surveyed = 1000

    Number of people naming the Bible as the most influential book = 999
    Number of people naming 1984 as the most influential book = 1
In this example, 1984 has come in second, but no one would say this second place finish supports a conclusion that “1984 has exercised much influence on a great number of this newspaper’s readers.” You can expect the correct answer to address this issue.

Answer choice (A): The survey in the argument asks readers to name the one book with the most influence in their lives; the number of books read does not affect this answer. To apply the Variance Test, try opposite answers of “1” and a large number, say “10,000.” These numbers will not alter the evaluation of the argument, and thus this answer is incorrect.

Answer choice (B): This is the correct answer choice, but it can be difficult since the wording is a bit unusual. The question is intended to reveal how many people selected 1984 relative to the other choices, and this addresses the issue raised in the analysis of the stimulus. Consider how the variance test works for this answer choice:


First try the response, “999.” In this case, only one person selected 1984 as the most influential book, and the argument is greatly weakened.

Next try the response, “501.” In this instance, 499 people selected 1984 as the most influential book and the conclusion is strengthened (the other 501 people would have selected the Bible). Note that you cannot try a number less than 501 because that would mean that the Bible was not named most often.

Because the varied responses produce different evaluations of the argument, this answer is correct.

Answer choice (C): This answer is not relevant to the columnist’s argument. Apply the Variance Test to disprove this answer by using opposite answers of “0” and a very large number, such as “1 million.”

Answer choice (D): Because the argument is about Orwell’s 1984, other Orwell books chosen by the readers have no impact on the argument. Apply the Variance Test, using opposite answers of “0” and a small number such as “10” (Orwell wrote dozens of essays, but not dozens of books).

Answer choice (E): The survey in the argument addresses influence, not the actual reading of the book. A person might be influenced by a book like the Bible through church teachings, etc. without actually having read the book. To apply the Variance Test, try opposite answers of “0” and “1000.”
 SherryZ
  • Posts: 124
  • Joined: Oct 06, 2013
|
#12762
Dec 2001, LSAT Sec 1 LR, Q24:

This is "Evaluation" question, could you tell me some techs to attack this type of question?

I chose C because first of all, I did not understand this question thoroughly, and secondly, I saw the stimulus says "the book has influence on THIS NEWSPAPER'S READERS".

The correct answer is B. Could you explain why B is right and C is wrong?

Thank you very much!

---Sherry
 Lucas Moreau
PowerScore Staff
  • PowerScore Staff
  • Posts: 216
  • Joined: Dec 13, 2012
|
#12826
Hello, Sherry,

Ugh, this question. This question is a real troublemaker. :x

Evaluate questions are tricky. They're asking you: what piece of information do you need to know in order to know whether or not the conclusion of this argument follows from its premises?

In this case, the conclusion of the argument is "George Orwell's book 1984 has exercised much influence on a great number of this newspaper's readers." Given the information provided about the survey, there is still one piece of information we need to know: exactly how did the numbers break down?

For Evaluate questions like this one, it can be useful to go to extremes. We know that the Bible was chosen first and 1984 was chosen second. With 1,000 readers surveyed, it is possible on one extreme - the most number of people that could have chosen books other than 1984 - for 999 people to have chosen the Bible and one person to have chosen 1984. This would not lead to the conclusion, since the percentage of people who chose 1984 in that instance would be negligibly small.

It could also be that, for instance, 3 people chose the Bible, 2 people chose 1984, and 995 people all chose a different book each. That would lead to the same result.

But what about the other extreme? What's the least number of people that could have chosen books other than 1984? Assuming the other premises are true, the least number is 501 people choosing the Bible to 499 people choosing 1984. This would lead to the conclusion, or would at least be strong evidence indicating the conclusion is true. The percentage of people choosing 1984 would be nearly half, which is certainly "a great number".

Hope that helps,
Lucas Moreau
PowerScore
 Mustafaabdulmalek
  • Posts: 19
  • Joined: Nov 17, 2015
|
#21967
what if the readers just read one book ?wouldn't this weaken the survey and the conclusion based on it accordingly because the survey would be bias
and for answer choice A I can't assume that the number of reader for the book is small because it said in the stimulus that r "on a GREAT number of newspaper's readers"
 Emily Haney-Caron
PowerScore Staff
  • PowerScore Staff
  • Posts: 577
  • Joined: Jan 12, 2012
|
#21977
Hi Mustafa,

This is a pretty tricky question, and most people struggle with it, so I'm glad you asked about it! Here's the thing: how much a book influences you has little or nothing to do with how many other books you've read. You could read one book and find it influential (or not) even if you had read no other books. Or you could read one million books but still find one (or some) of them very influential. So answer A is irrelevant. B, on the other hand, is important to figuring out whether "a great number" of readers really were influenced. If 999 readers chose the Bible, and one person chose 1984, then that's not a lot of influence! But if 501 people chose the Bible, and 499 chose 1984, then that actually supports the claim that it exercised influence on "a great number" of readers.

Does that help?
 rhjones2691
  • Posts: 17
  • Joined: Feb 07, 2018
|
#45389
Hello,

I understand the logic behind answer choice (B), and why that is the correct answer. However, I am unsure why (E) is an incorrect answer. Applying the variance test, if 0% of readers actually read the books they selected (and thus arbitrarily selected), that would seem to weaken the conclusion that the books exercised much influence on the readers. Conversely, if 100% of readers read the books they chose, it would make the conclusion more likely to be true (albeit to less of a degree than 0% weakens the conclusion). I suppose that an argument could be made that even in a random selection, the selection of an item itself means that some influence in some form has been made upon the reader (playing off of the unclear nature of "influence"). Perhaps this is simply an instance of the best answer choice, since (E) does still seem to require some knowledge of the distribution of book selection. Can you please help clarify this? Thanks!
 Alex Bodaken
PowerScore Staff
  • PowerScore Staff
  • Posts: 136
  • Joined: Feb 21, 2018
|
#45394
rhjones2691,

This is an excellent question. I think you get towards the reasoning at the end of your question...basically, this is an instance of (B) being an excellent answer choice, and (E) being somewhat less so. Applying the variance test to (B) would yield at one extreme that 501 people chose the Bible and 499 chose 1984 (with 0 choosing others)...which would strongly support the authors conclusion; at the other extreme, it would mean that 999 people chose the Bible and 1 person chose 1984, which would strongly weaken the author's argument. So (B) is a very strong answer.

(E) is less strong precisely because of the reason you noted - it is certainly possible to be influenced by a book that you haven't read. The Bible, incidentally, is a pretty good example of this...I would guess that the numbers of people who have been influenced by the Bible is many orders of magnitude of folks larger than those who have actually read it cover to cover. I say this simply to note that it's clear one can be influenced by a book without having read it. Therefore, even if (for example) 0% of those who selected 1984 had read it, it is absolutely possible that they could have been influenced by it. This is the weakness of (E) as an answer choices and why (B) is credited.

Hope that helps!
Alex
 rhjones2691
  • Posts: 17
  • Joined: Feb 07, 2018
|
#45523
Thanks, Alex! That clarification helps.
 lsatnoobie
  • Posts: 52
  • Joined: Sep 18, 2017
|
#46162
Hi Powerscore,

I’m still unsure as to why C is incorrect (I do understand why B is right). Answers to an evaluate question must be able to both strengthen and weaken the argument. For C, couldn’t I strengthen it by saying only 1,000 people read the newspaper, thus making the sample representation 100%. Similarly, can’t I weaken the argument by saying 7 Billion people read the newspaper (not a representative sample, thus we don’t know whether 1984 had a huge influence).

All other answers tell me 1,000 is enough of a sample size but that is not a convincing answer because who determines that?

Any help would be appreciated thank you.
User avatar
 Jonathan Evans
PowerScore Staff
  • PowerScore Staff
  • Posts: 726
  • Joined: Jun 09, 2016
|
#46764
Hi, lsatnoobie,

Good job using the Variance Test™ with answer choice (C). It is possible that you could devise divergent scenarios with answer choice (C) to strengthen or weaken the argument. It is true that the argument would be stronger if the total readership were the same 1,000 people who responded to the survey. It is likewise true that the argument would be weaker if the total readership were 500 million.

However, even if the readership were extraordinarily large, we have no additional reason to believe this sample is unrepresentative. There is nothing in the stimulus to suggest that the respondents are a self-selected group. There is no evidence in the stimulus to suggest these respondents are in any way different from the readership at large. Thus, while a very large readership would somewhat call into question the columnist's conclusion, the size of the sample does not in and of itself establish that the sample is unrepresentative.

Answer choice (B) in contrast gives a direct Variance Test that results either in a much stronger or much weaker argument. Since we must select the best response, the question that would be most helpful in evaluating the argument, we should choose (B) since it has the most direct impact on the validity of the argument.

I hope this helps!

Get the most out of your LSAT Prep Plus subscription.

Analyze and track your performance with our Testing and Analytics Package.